Diễn Đàn MathScopeDiễn Đàn MathScope
  Diễn Đàn MathScope
Ghi Danh Hỏi/Ðáp Community Lịch

Go Back   Diễn Đàn MathScope > Sơ Cấp > Đại Số và Lượng Giác > Các Bài Toán Đã Được Giải

News & Announcements

Ngoài một số quy định đã được nêu trong phần Quy định của Ghi Danh , mọi người tranh thủ bỏ ra 5 phút để đọc thêm một số Quy định sau để khỏi bị treo nick ở MathScope nhé !

* Nội quy MathScope.Org

* Một số quy định chung !

* Quy định về việc viết bài trong diễn đàn MathScope

* Nếu bạn muốn gia nhập đội ngũ BQT thì vui lòng tham gia tại đây

* Những câu hỏi thường gặp

* Về việc viết bài trong Box Đại học và Sau đại học


Ðề tài đã khoá Gởi Ðề Tài Mới
 
Ðiều Chỉnh Xếp Bài
Old 20-07-2011, 02:06 PM   #151
bboy114crew
+Thành Viên+
 
Tham gia ngày: Oct 2010
Đến từ: Dòng thời gian...
Bài gởi: 294
Thanks: 290
Thanked 189 Times in 91 Posts
Gửi tin nhắn qua Yahoo chát tới bboy114crew
Cho a, b, c dương có tổng bằng 1. Chứng minh rằng
$\frac{a^4}{a^3+3b^3}+\frac{b^4}{b^3+3c^3}+\frac{c^ 4}{c^3+3a^3}\geq \frac{1}{4} $
[RIGHT][I][B]Nguồn: MathScope.ORG[/B][/I][/RIGHT]
 
__________________
Thay đổi tất cả và mãi mãi......
Offline...
bboy114crew is offline  
The Following User Says Thank You to bboy114crew For This Useful Post:
nguyenthaovan (22-07-2011)
Old 20-07-2011, 02:18 PM   #152
caoanhtuyet
+Thành Viên+
 
Tham gia ngày: Jul 2011
Bài gởi: 1
Thanks: 0
Thanked 1 Time in 1 Post
Trích:
Nguyên văn bởi bboy114crew View Post
Cho a, b, c dương có tổng bằng 1. Chứng minh rằng
$\frac{a^4}{a^3+3b^3}+\frac{b^4}{b^3+3c^3}+\frac{c^ 4}{c^3+3a^3}\geq \frac{1}{4} $
1. Ý tưởng đầu tiên của mình là dùng Cauchy-scharwz
2. Dùng AM-GM
Ta có :$\frac{a^4}{a^3+3b^3}=a-\frac{3ab^3}{a^3+3b^3}\ge a-\frac{3\sqrt[4]{ab^3}}{4}\ge \frac{13a-9b}{16} $
Tiếp tục xây dựng như thế, sau đó cộng các bất đẳng thức lại ta suy ra điều phải chứng minh
[RIGHT][I][B]Nguồn: MathScope.ORG[/B][/I][/RIGHT]
 

thay đổi nội dung bởi: caoanhtuyet, 20-07-2011 lúc 02:33 PM
caoanhtuyet is offline  
The Following User Says Thank You to caoanhtuyet For This Useful Post:
Katyusha (21-07-2011)
Old 21-07-2011, 06:47 PM   #153
Conan Edogawa
+Thành Viên+
 
Conan Edogawa's Avatar
 
Tham gia ngày: Sep 2008
Đến từ: Trường ĐH Kinh tế TP.HCM
Bài gởi: 397
Thanks: 136
Thanked 303 Times in 150 Posts
Cho $ a,b,c\ge 0 $ thỏa mãn không có 2 số nào trong chúng đồng thời bằng 0.

CM: $\frac{a}{\sqrt{4{{b}^{2}}+bc+4{{c}^{2}}}}+\frac{b} {\sqrt{4{{c}^{2}}+ca+4{{a}^{2}}}}+ \frac{c}{\sqrt{4{{a}^{2}}+ab+4{{b}^{2}}}}\ge 1 $
[RIGHT][I][B]Nguồn: MathScope.ORG[/B][/I][/RIGHT]
 
Conan Edogawa is offline  
Old 21-07-2011, 09:10 PM   #154
Katyusha
+Thành Viên+
 
Tham gia ngày: Apr 2011
Bài gởi: 180
Thanks: 134
Thanked 21 Times in 21 Posts
Mọi người giúp em bài này ạ

Trích:
Cho $x,y,z \geq 0; x+y+z=1 $. Tìm GTLN của:
$P=x^ny+y^nz+z^nx $ với $n \in N* $

[RIGHT][I][B]Nguồn: MathScope.ORG[/B][/I][/RIGHT]
 
Katyusha is offline  
Old 21-07-2011, 09:37 PM   #155
manhnguyen94
+Thành Viên+
 
Tham gia ngày: Dec 2009
Đến từ: 11 Toán CQB
Bài gởi: 98
Thanks: 83
Thanked 69 Times in 38 Posts
Trích:
Nguyên văn bởi Conan Edogawa View Post
Cho $ a,b,c\ge 0 $ thỏa mãn không có 2 số nào trong chúng đồng thời bằng 0.

CM: $\frac{a}{\sqrt{4{{b}^{2}}+bc+4{{c}^{2}}}}+\frac{b} {\sqrt{4{{c}^{2}}+ca+4{{a}^{2}}}}+ \frac{c}{\sqrt{4{{a}^{2}}+ab+4{{b}^{2}}}}\ge 1 $
CS:
$Vt \ge \frac{(a+b+c)^2}{a\sqrt{4b^2+bc+c^2}+b\sqrt{4a^2+a c+4c^2}+c\sqrt{4a^2+ab+4b^2}} $
Lại áp dụng CS :
$(a\sqrt{4b^2+bc+4c^2}+b{\sqrt{4{{c}^{2}}+ca+4{{a}^ {2}}}+c\sqrt{4{{a}^{2}}+ab+4{{b}^{2}}})^2 \le (a+b+c)(4 \sum ab(a+b) +3abc) \le (a+b+c)(a^3+b^3+c^3 +3 \sum ab(a+b)+6abc) =(a+b+c)^4 $
=> đpcm
[RIGHT][I][B]Nguồn: MathScope.ORG[/B][/I][/RIGHT]
 

thay đổi nội dung bởi: manhnguyen94, 21-07-2011 lúc 09:41 PM
manhnguyen94 is offline  
The Following 3 Users Say Thank You to manhnguyen94 For This Useful Post:
bboy114crew (21-07-2011), Conan Edogawa (21-07-2011), nguyenthaovan (22-07-2011)
Old 21-07-2011, 09:44 PM   #156
khtoan
+Thành Viên+
 
Tham gia ngày: Jan 2010
Đến từ: Đà Nẵng
Bài gởi: 155
Thanks: 23
Thanked 128 Times in 68 Posts
Trích:
Nguyên văn bởi Conan Edogawa View Post
Cho $ a,b,c\ge 0 $ thỏa mãn không có 2 số nào trong chúng đồng thời bằng 0.

CM: $\frac{a}{\sqrt{4{{b}^{2}}+bc+4{{c}^{2}}}}+\frac{b} {\sqrt{4{{c}^{2}}+ca+4{{a}^{2}}}}+ \frac{c}{\sqrt{4{{a}^{2}}+ab+4{{b}^{2}}}}\ge 1 $
Bài này Holder cũng ngon mà

Đặt $S=\frac{a}{\sqrt{4{{b}^{2}}+bc+4{{c}^{2}}}}+\frac{ b}{\sqrt{4{{c}^{2}}+ca+4{{a}^{2}}}}+ \frac{c}{\sqrt{4{{a}^{2}}+ab+4{{b}^{2}}}} $

Theo Holder:

$S^2(\sum a(4b^2+bc+4c^2))\geq (a+b+c)^3
$

Vậy ta chỉ cần đi chứng minh:

$(a+b+c)^3\geq \sum a(4b^2+bc+4c^2) $

Khai triển và rút gọn ta được 1 bất đẳng thức đúng .
[RIGHT][I][B]Nguồn: MathScope.ORG[/B][/I][/RIGHT]
 
khtoan is offline  
The Following 5 Users Say Thank You to khtoan For This Useful Post:
bboy114crew (21-07-2011), Conan Edogawa (21-07-2011), hoangcongduc (24-07-2011), mrvui123 (04-03-2012), nguyenthaovan (22-07-2011)
Old 21-07-2011, 09:57 PM   #157
bboy114crew
+Thành Viên+
 
Tham gia ngày: Oct 2010
Đến từ: Dòng thời gian...
Bài gởi: 294
Thanks: 290
Thanked 189 Times in 91 Posts
Gửi tin nhắn qua Yahoo chát tới bboy114crew
Thêm hai bài nữa!
1) Cho a,b,c > 0, $abc \geq 1 $.CMR:
$\frac{a}{\sqrt{b+\sqrt{ac}}} + \frac{b}{\sqrt{c+\sqrt{ab}}} $$+\frac{c}{\sqrt{a+\sqrt{bc}}} \geq \frac{3}{\sqrt{2}} $
2)Cho $a,b,c \geq 1 $.CMR:
$\sum a(b+c)+\sum\frac{1}{1+a^2} \geq \frac{15}{2} $
[RIGHT][I][B]Nguồn: MathScope.ORG[/B][/I][/RIGHT]
 
__________________
Thay đổi tất cả và mãi mãi......
Offline...

thay đổi nội dung bởi: bboy114crew, 22-07-2011 lúc 09:33 AM
bboy114crew is offline  
The Following User Says Thank You to bboy114crew For This Useful Post:
nguyenthaovan (22-07-2011)
Old 21-07-2011, 10:18 PM   #158
khtoan
+Thành Viên+
 
Tham gia ngày: Jan 2010
Đến từ: Đà Nẵng
Bài gởi: 155
Thanks: 23
Thanked 128 Times in 68 Posts
Trích:
Nguyên văn bởi bboy114crew View Post
Thêm hai bài nữa!
1) Cho a,b,c > 0, $abc \geq 1 $.CMR:
$\frac{a}{\sqrt{b+\sqrt{ac}}} + \frac{b}{\sqrt{c+\sqrt{ab}}} $$+\frac{c}{\sqrt{a+\sqrt{bc}}} \geq \frac{3}{\sqrt{2}} $
2)Cho $a,b,c \geq 1 $.CMR:
$\sum a(b+c)+\sum\frac{1}{1+a^2} \geq \frac{15}{2} $
Bài 1 :
Áp dụng BĐT CS và AM - GM ta có
$A\ge \frac{{{(\sqrt{a}+\sqrt{b}+\sqrt{c})}^{2}}}{\sqrt{ b+\sqrt{ac}}+\sqrt{c+\sqrt{ab}}+\sqrt{a+\sqrt{bc}} }=\frac{2\sqrt{2}{{(\sqrt{a}+\sqrt{b}+\sqrt{c})}^{ 2}}}{2\sqrt{2}\sqrt{b+\sqrt{ac}}+2\sqrt{2}\sqrt{c+ \sqrt{ab}}+2\sqrt{2}\sqrt{a+\sqrt{bc}}} $
$\ge \frac{2\sqrt{2}{{\left( \sqrt{a}+\sqrt{b}+\sqrt{c} \right)}^{2}}}{2+b+\sqrt{ac}+2+c+\sqrt{ab}+2+a+\sq rt{bc}}=\frac{2\sqrt{2}{{\left( \sqrt{a}+\sqrt{b}+\sqrt{c} \right)}^{2}}}{a+b+c+\sqrt{ab}+\sqrt{bc}+\sqrt{ca} +6} $

Ta cần chứng minh :

$\frac{2\sqrt{2}{{\left( \sqrt{a}+\sqrt{b}+\sqrt{c} \right)}^{2}}}{a+b+c+\sqrt{ab}+\sqrt{bc}+\sqrt{ca} +6}\ge \frac{3}{\sqrt{2}} $
$\Leftrightarrow 4\left( a+b+c+2\sqrt{ab}+2\sqrt{bc}+2\sqrt{ca} \right)\ge 3\left( a+b+c+\sqrt{ab}+\sqrt{bc}+\sqrt{ca} \right)+18 $
$\Leftrightarrow a+b+c+5\left( \sqrt{ab}+\sqrt{bc}+\sqrt{ca} \right)\ge 18 $

BĐT cuối này đúng do $a+b+c\ge 3\sqrt[3]{abc}\ge 3 $
$5\left( \sqrt{ab}+\sqrt{bc}+\sqrt{ca} \right)\ge 15\sqrt[3]{\sqrt{ab}.\sqrt{bc}.\sqrt{ca}}=15\sqrt[3]{abc}\ge 15 $

Ta có đpcm

Bài 2
Theo giả thiết $a,b,c\geq 1 $ ta có :

$VT\geq \frac{3}{4}\sum a(b+c)+\frac{a(b+c)}{4}+\frac{b(a+c)}{4}+\frac{c(a +b)}{4}+\frac{1}{ab+a^2}+\frac{1}{bc+b^2}+\frac{1} {ca+a^2} $

Theo AM-GM 6 số :

$\frac{a(b+c)}{4}+\frac{b(a+c)}{4}+\frac{c(a+b)}{4} +\frac{1}{ab+a^2}+\frac{1}{bc+b^2}+\frac{1}{ca+a^2 }\geq 3 $

và $\frac{3}{4}\sum a(b+c)\geq \frac{9}{2} $

Từ đó có $S\geq \frac{15}{2} $


[RIGHT][I][B]Nguồn: MathScope.ORG[/B][/I][/RIGHT]
 

thay đổi nội dung bởi: khtoan, 21-07-2011 lúc 10:44 PM
khtoan is offline  
The Following 2 Users Say Thank You to khtoan For This Useful Post:
bboy114crew (22-07-2011), nguyenthaovan (22-07-2011)
Old 23-07-2011, 09:56 AM   #159
chém gà
+Thành Viên+
 
Tham gia ngày: Jul 2011
Đến từ: France
Bài gởi: 136
Thanks: 8
Thanked 60 Times in 54 Posts
Trích:
Nguyên văn bởi bboy114crew View Post
Thêm hai bài nữa!
1) Cho a,b,c > 0, $abc \geq 1 $.CMR:
$\frac{a}{\sqrt{b+\sqrt{ac}}} + \frac{b}{\sqrt{c+\sqrt{ab}}} $$+\frac{c}{\sqrt{a+\sqrt{bc}}} \geq \frac{3}{\sqrt{2}} $
Bài này sao không C-S với Côsi thế này nhìn cho ác nhỉ:
$\sum \frac{a}{\sqrt{b+\sqrt{ac}}} \ge \frac{(a+b+c)^2}{\sum a\sqrt{b+\sqrt{ac}}} \ge \frac{(a+b+c)^2}{\sqrt{(a+b+c)(ab+bc+ca+a\sqrt{ac} +b\sqrt{ab}+ c\sqrt{bc})}} $
$\ge \frac{(a+b+c)^2}{\sqrt{(a+b+c)(ab+bc+ca+\frac{a^2+ ac}{2}+\frac{b^2+ab}{2}+\frac{c^2+bc}{2})}}=\frac{ (a+b+c)^2}{\sqrt{(a+b+c)(\frac{(a+b+c)^2+ab+bc+ca} {2})}} $
$\ge \frac{(a+b+c)^2}{\sqrt{\frac{2(a+b+c)^3}{3}}}= \sqrt{\frac{3(a+b+c)}{2}} \ge \frac{3}{\sqrt{2}} $
[RIGHT][I][B]Nguồn: MathScope.ORG[/B][/I][/RIGHT]
 
__________________
Gà nhất quả đất-Nát nhất Sever

chém gà is offline  
The Following User Says Thank You to chém gà For This Useful Post:
bboy114crew (24-07-2011)
Old 23-07-2011, 11:22 AM   #160
Uy_Vũ
+Thành Viên+
 
Uy_Vũ's Avatar
 
Tham gia ngày: Jan 2010
Đến từ: Dân tộc Mường
Bài gởi: 128
Thanks: 8
Thanked 68 Times in 40 Posts
BDT này đúng ko ?
Cho $a,b,c>0; a+b+c=3 $.CMR:
$\sum \frac{1}{\sqrt[3]{a^2-a+1}} \le 3 $
[RIGHT][I][B]Nguồn: MathScope.ORG[/B][/I][/RIGHT]
 
__________________
Giang hồ nổi gió từ đây.
Chuyên Anh
Uy_Vũ is offline  
Old 23-07-2011, 11:27 AM   #161
Hoang Nguyen
+Thành Viên+
 
Tham gia ngày: Jun 2011
Bài gởi: 77
Thanks: 25
Thanked 6 Times in 5 Posts
1.Cho a,b,c là các số thực dương thỏa mãn a+b+c=3.Chứng minh bất đẳng thức sau
$\sqrt{\frac{a+b}{c+ab}}+\sqrt{\frac{b+c}{a+bc}}+ \sqrt{\frac{c+a}{b+ca}}\ge 3 $
2.Cho a,b,c >0 Chứng minh rằng
$\frac{a^2+b^2+c^2}{ab+bc+ca}+\frac{1}{3}\ge \frac{8}{9}(\frac{a}{c+b}+\frac {b}{a+c}+\frac{c}{a+b}) $
[RIGHT][I][B]Nguồn: MathScope.ORG[/B][/I][/RIGHT]
 
Hoang Nguyen is offline  
Old 23-07-2011, 11:47 AM   #162
khtoan
+Thành Viên+
 
Tham gia ngày: Jan 2010
Đến từ: Đà Nẵng
Bài gởi: 155
Thanks: 23
Thanked 128 Times in 68 Posts
Trích:
Nguyên văn bởi Hoang Nguyen View Post
1.Cho a,b,c là các số thực dương thỏa mãn a+b+c=3.Chứng minh bất đẳng thức sau
$\sqrt{\frac{a+b}{c+ab}}+\sqrt{\frac{b+c}{a+bc}}+ \sqrt{\frac{c+a}{b+ca}}\ge 3 $
2.Cho a,b,c >0 Chứng minh rằng
$\frac{a^2+b^2+c^2}{ab+bc+ca}+\frac{1}{3}\ge \frac{8}{9}(\frac{a}{c+b}+\frac {b}{a+c}+\frac{c}{a+b}) $
Bài 1 đã có ở đây
[Only registered and activated users can see links. ]

Bài 2: Ta sẽ đi chứng minh bất đẳng thức sau:

$\frac{a^2}{ab+bc+ca}+\frac{1}{9}\geq \frac{8}{9}\frac{a}{b+c} $

Thật vậy ,điều đó tương đương với :

$\frac{a^2}{ab+bc+ca}+1\geq \frac{8}{9}\frac{a+b+c}{b+c}\Leftrightarrow \frac{9(a+b)(a+c)}{ab+bc+ca}\geq \frac{8(a+b+c)}{b+c}\Leftrightarrow 9(a+b)(b+c)(c+a)\geq 8(a+b+c)(ab+bc+ca) $ (Đúng vì đây là 1 bdt quen thuộc )

Làm 2 cái còn lại rồi cộng vế theo vế ta có đpcm.

__________________________________________________ _____________________________
Mọi người thử làm bài này

Cho a,b,c là các số thực dương.Chứng minh rằng:

1) $\frac{a^2+b^2+c^2}{ab+bc+ca}\geq \frac{a(b+c)}{a^2+5bc}+\frac{b(c+a)}{b^2+5ca}+ \frac{c(a+b)}{c^2+5ab} $

2) $64(a^2+bc)(b^2+ac)(c^2+ab)\leq (a+b+c)^6 $
[RIGHT][I][B]Nguồn: MathScope.ORG[/B][/I][/RIGHT]
 

thay đổi nội dung bởi: khtoan, 23-07-2011 lúc 08:39 PM
khtoan is offline  
The Following User Says Thank You to khtoan For This Useful Post:
Ngô_Trung_Hiếu (23-07-2011)
Old 23-07-2011, 11:14 PM   #163
_minhhoang_
+Thành Viên+
 
Tham gia ngày: Sep 2010
Đến từ: Saint Petersburg
Bài gởi: 126
Thanks: 18
Thanked 221 Times in 75 Posts
Gửi tin nhắn qua Yahoo chát tới _minhhoang_
Trích:
Nguyên văn bởi khtoan View Post
2) $64(a^2+bc)(b^2+ac)(c^2+ab)\leq (a+b+c)^6 $
Không mất tính tổng quát, giả sử c là số nhỏ nhất
Ta sẽ chứng minh :
$ 64({a^2} + bc)({b^2} + ac)({c^2} + ab) \le 16{(\frac{{{{(a + b)}^2}}}{2} + c(a + b))^2}({c^2} + \frac{{{{(a + b)}^2}}}{4}) $
Bất đẳng thức trên đúng vì ta có thể dễ dàng chứng minh được:
$4({a^2} + bc)({b^2} + ac) \le {(\frac{{{{(a + b)}^2}}}{2} + c(a + b))^2} $
Và ${c^2} + ab \le {c^2} + \frac{{{{(a + b)}^2}}}{4} $
Do đó, theo BDT AM-GM được:
$ 16{(\frac{{{{(a + b)}^2}}}{2} + c(a + b))^2}({c^2} + \frac{{{{(a + b)}^2}}}{4}) $
$ = {({(a + b)^2} + 2c(a + b))^2}(4{c^2} + {(a + b)^2}) $
$ \le \frac{{{{\left( {3{{(a + b)}^2} + 4c(a + b) + 4{c^2}} \right)}^3}}}{{27}} $
$ \le \frac{{{{\left( {3{{(a + b)}^2} + 6c(a + b) + 3{c^2}} \right)}^3}}}{{27}} $
$ = {(a + b + c)^6} $
Bất đẳng thức được chứng minh, dấu "=" xảy ra khi (a,b,c)=(k,k,0) và các hoán vị
[RIGHT][I][B]Nguồn: MathScope.ORG[/B][/I][/RIGHT]
 
__________________
Хоанг
_minhhoang_ is offline  
The Following User Says Thank You to _minhhoang_ For This Useful Post:
company (24-07-2011)
Old 23-07-2011, 11:45 PM   #164
MathForLife
+Thành Viên+
 
Tham gia ngày: Sep 2010
Đến từ: CT force
Bài gởi: 731
Thanks: 603
Thanked 425 Times in 212 Posts
Trích:
Nguyên văn bởi khtoan View Post

2) $64(a^2+bc)(b^2+ac)(c^2+ab)\leq (a+b+c)^6 $
Không mất tính tổng quát, giả sử $a+b+c=2 $
Cần chứng minh:

$(a^2+bc)(b^2+ac)(c^2+ab)\le 1 $

Ta sẽ chứng minh bất đẳng thức mạnh hơn:

$\sum (a^2+bc)(b^2+ca)\le 3 $

$\Leftrightarrow \sum a^2b^2+(ab+bc+ca)(a^2+b^2+c^2)\le 3 $

Không mất tính tổng quát, giả sử c nhỏ nhất. Áp dụng bất đẳng thức AM-GM:

$2\sum a^2b^2\le ab(a^2+b^2)+2c^2(a^2+b^2)=(2c^2+ab)(a^2+b^2) $

$\le \frac{(a^2+b^2+4c^2+2ab)^2}{8}=\frac{(4c^2+(2-c)^2)^2}{8}=\frac{(4-c(4-5c))^2}{8}\le 2 $

Do đó $\sum a^2b^2\le 1 $

Lại theo bất đẳng thức AM-GM:

$(ab+bc+ca)(a^2+b^2+c^2)\le \frac{(a^2+b^2+c^2+2(ab+bc+ca))^2}{8}=2 $

Kết hợp các bất đẳng thức trên ta có đpcm.
[RIGHT][I][B]Nguồn: MathScope.ORG[/B][/I][/RIGHT]
 

thay đổi nội dung bởi: MathForLife, 24-07-2011 lúc 09:42 PM
MathForLife is offline  
The Following User Says Thank You to MathForLife For This Useful Post:
khtoan (24-07-2011)
Old 24-07-2011, 08:58 AM   #165
bboy114crew
+Thành Viên+
 
Tham gia ngày: Oct 2010
Đến từ: Dòng thời gian...
Bài gởi: 294
Thanks: 290
Thanked 189 Times in 91 Posts
Gửi tin nhắn qua Yahoo chát tới bboy114crew
1/Cho a,b,c>0 thỏa mãn a+b+c=3.CMR
$\sqrt {\frac{{a + 1}}{{{b^2} + 1}}} + \sqrt {\frac{{b + 1}}{{{c^2} + 1}}} + \sqrt {\frac{{c + 1}}{{{a^2} + 1}}} \ge 3 $
2/Cho a,b,c,d>0 thỏa mãn abcd=1.CMR
$\sqrt {\frac{{ab + 1}}{{a + 1}}} + \sqrt {\frac{{bc + 1}}{{b + 1}}} + \sqrt {\frac{{cd + 1}}{{c + 1}}} + \sqrt {\frac{{da + 1}}{{d + 1}}} \ge 4 $
[RIGHT][I][B]Nguồn: MathScope.ORG[/B][/I][/RIGHT]
 
__________________
Thay đổi tất cả và mãi mãi......
Offline...
bboy114crew is offline  
Ðề tài đã khoá Gởi Ðề Tài Mới

Bookmarks


Quuyền Hạn Của Bạn
You may not post new threads
You may not post replies
You may not post attachments
You may not edit your posts

BB code is Mở
Smilies đang Mở
[IMG] đang Mở
HTML đang Tắt

Chuyển đến


Múi giờ GMT. Hiện tại là 02:42 PM.


Powered by: vBulletin Copyright ©2000-2024, Jelsoft Enterprises Ltd.
Inactive Reminders By mathscope.org
[page compression: 106.00 k/122.38 k (13.38%)]